How Do You Calculate Surface Integrals for Vector Fields?

Click For Summary
The discussion focuses on calculating surface integrals for vector fields using Stoke's theorem. The first problem involves evaluating the flux of a constant vector field through a hemispherical surface, where the curl is zero, leading to a flux of zero without needing to evaluate the surface integral. The second problem addresses a vector field defined by \(\vec A = y\hat i + z\hat j + x\hat k\) and requires finding the surface integral over a paraboloid, with the curl calculated as \(-1(\hat i + \hat j + \hat k)\). The procedure involves determining the normal vector to the surface and integrating over the projected region in the xy-plane, which is a circle for \(z > 0\). The final evaluation of the integral yields a result of -3.
Reshma
Messages
749
Reaction score
6
I have two problems on surface integrals.

1] I have a constant vector \vec v = v_0\hat k. I have to evaluate the flux of this vector field through a curved hemispherical surface defined by x^2 + y^2 + z^2 = r^2, for z>0. The question says use Stoke's theorem.

Stoke's theorem suggests:
\int_s \left(\vec \nabla \times \vec v\right) \cdot d\vec a = \int_p \vec v \cdot d\vec l

But the curl of this vector comes out to be zero . Am I going right? How is the surface integral evaluated?

2] I have a vector field \vec A = y\hat i + z\hat j + x\hat k. I have to find the value of the surface integral:
\int_s \left(\vec \nabla \times \vec A\right) \cdot d\vec a

The surface S here is a paraboloid defined by:
z = 1 - x^2 - y^2

I evaluated the curl and it comes out to be:
\vec \nabla \times \vec A = -1\left(\hat i + \hat j + \hat k\right)

I need help here on the procedure to evaluate the surface integral.
 
Last edited:
Physics news on Phys.org
Reshma said:
I have two problems on surface integrals.

1] I have a constant vector \vec v = v_0\hat k. I have to evaluate the flux of this vector field through a curved hemispherical surface defined by x^2 + y^2 + z^2 = r^2, for z>0. The question says use Stoke's theorem.

Stoke's theorem suggests:
\int_s \left(\vec \nabla \times \vec v\right) \cdot d\vec a = \int_p \vec v \cdot d\vec l

But the curl of this vector comes out to be zero . Am I going right? How is the surface integral evaluated?
Not , :biggrin: ! The integral of 0 over any volume is 0 so you have the answer right in front of you! Since you are using Stoke's theorem, you don't need to evaluate the surface integral.

2] I have a vector field \vec A = y\hat i + z\hat j + x\hat k. I have to find the value of the surface integral:
\int_s \left(\vec \nabla \times \vec A\right) \cdot d\vec a

The surface S here is a paraboloid defined by:
z = 1 - x^2 - y^2

I evaluated the curl and it comes out to be:
\vec \nabla \times \vec A = -1\left(\hat i + \hat j + \hat k\right)

I need help here on the procedure to evaluate the surface integral.
You probably have a formula for d\vec a but here is how I like to think about it: The surface is given by z= 1- x2- y2 which is the same as x2+ y2+ z= 1. We can think of that as a "level surface" of the function f(x,y,z)= x2+y2+ z. The gradient of that: 2xi+ 2yj+ k is normal to the surface and, since it is "normalized" to the xy-plane in the sense that the k component is 1, d\vec a= (2xi+ 2yj+ k)dxdy.
Take the dot product of that with -(i+ j+ k) to get the integrand. You don't say over what region of the paraboloid that is to be integrated. If it is over the region above z= 0, then projected into the xy-plane, you have the circle x2+ y2= 1. Integrate over that circle. It will probably be simplest to do it in polar coordinates.
 
Last edited by a moderator:
Thank you very, very much! o:)

\left(\vec \nabla \times \vec A\right) \cdot d\vec a = -\left(2x + 2y + 1\right)dxdy

The region of integration is over z>0. Just a little doubt...will the projected region on the xy-plane be a circle even if z>0?

\int_s \left(\vec \nabla \times \vec A\right) \cdot d\vec a = -\int_0^1 \int_0^1 \left(2x + 2y + 1\right)dxdy

\int_s \left(\vec \nabla \times \vec A\right) \cdot d\vec a = -\int_0^1 \int_0^1 \left(2xdx + 2ydx + 1dx\right)dy

\int_s \left(\vec \nabla \times \vec A\right) \cdot d\vec a = -2\int_0^1 (1+y)dy

Evaluation of this gives me: -3
 
Last edited:
Question: A clock's minute hand has length 4 and its hour hand has length 3. What is the distance between the tips at the moment when it is increasing most rapidly?(Putnam Exam Question) Answer: Making assumption that both the hands moves at constant angular velocities, the answer is ## \sqrt{7} .## But don't you think this assumption is somewhat doubtful and wrong?

Similar threads

  • · Replies 9 ·
Replies
9
Views
2K
  • · Replies 9 ·
Replies
9
Views
1K
  • · Replies 14 ·
Replies
14
Views
2K
  • · Replies 7 ·
Replies
7
Views
2K
  • · Replies 4 ·
Replies
4
Views
2K
  • · Replies 4 ·
Replies
4
Views
2K
Replies
3
Views
2K
Replies
6
Views
2K
Replies
20
Views
2K
  • · Replies 6 ·
Replies
6
Views
3K